Calcolo 1 Domanda pratica - A

Una serie di domande di Calcolo 1 con le relative soluzioni dettagliate per esercitarsi per test, esami, esami di collocamento... e acquisire una comprensione approfondita dei seguenti argomenti:

  1. Funzioni
  2. Limiti
  3. Continuità
  4. Derivati
  5. Applicazioni di derivati
Le domande sono state progettate per coprire gli argomenti più importanti nel calcolo 1 e le loro soluzioni dettagliate includono collegamenti a più pratica in questi argomenti.


  1. Domanda 1
    Calcola il dominio della funzione Function in Question 1

  2. Domanda 2
    Calcolare l'intervallo della funzione Function in Question 2 \( \)\( \)\( \)\( \)\( \)

  3. Domanda 3
    Trova l'inverso della funzione \( f(x) = ln (2x - 3) + 2 \). Function in Question 3

  4. Domanda 4
    Valuta ciascuno dei seguenti limiti
    Limit in Question 4 a

    Limit in Question 4 b

    Limit in Question 4 c

    Limit in Question 4 d

    Limit in Question 4 e

  5. Domanda 5
    a) Disegna \( y = e^{x-1} \) e \( y = x \) nello stesso sistema di coordinate e poi mostra che i due grafici sono tangenti nel punto \( (1,1) \ ) e che \( e^{x-1} \ge x \)
    b) Utilizzare il risultato ottenuto nella parte a) per determinare la concavità della funzione \( f(x) = \dfrac{x^3}{6} - e^{x-1} \) e gli eventuali punti di flesso.

  6. Domanda 6
    Trova la derivata delle seguenti funzioni e non semplificare la risposta finale.

    a) \( f(x) = e^{x-1} + \ln (3x-1) + \sin(2x+1) \)

    b) \( g(x) = (2x-1)^2(\tan(x)-1) \)

    c) \( h(x) = \dfrac{x - \cos(x)}{x^2-2x+1} \)

    d) \( m(x) = \sin \left(\sqrt{x^3 - \dfrac{1}{x} + 2} \right) \)

    e) \( n(x) = 3^{ 2x+3} + \log_3(2x-1) \)

  7. Domanda 7
    Trova l'equazione della retta tangente alla curva con l'equazione \( \sin(y^2) = x^2 \) nel punto \( ( 0,\sqrt{\pi}) \).

  8. Domanda 8
    Trova le costanti \( a \) e \( b \) in modo che la funzione \( f \) sia continua su \( (-\infty , +\infty ) \)
    \( f(x) = \left\{ \begin{array}{ll} 2x - 1 & x\le 1 \\ a x^3 + b & 1 \lt x \lt 2 \\ x + 2 b & x\ge 2 \\ \end{array} \right. \)

  9. Domanda 9
    Trova l'equazione della retta tangente alla curva con l'equazione \( y = x + \sin(x) \) in \( x = 0 \).

  10. Domanda 10
    Usa la definizione della derivata come limite per trovare la derivata \( f' \) dove \( f(x) = \sqrt{x+2} \).

  11. Domanda 11
    Determina su quale/i intervallo/i è la funzione \( f(x) = e^x(x^2-5x+8)+\dfrac{x^4}{12}-\dfrac{x^3}{6 } \) concava verso l'alto e concava verso il basso e qualsiasi punto di flesso.

  12. Domanda 12
    Utilizzare il metodo di Newton con approssimazione iniziale \( x_1 = 2 \) per trovare una seconda approssimazione alla soluzione dell'equazione \( e^x = x^3 \).

  13. Domanda 13
    Trova il massimo e il minimo assoluti della funzione \( f(x) = x^4 - x^3 \) sull'intervallo \( [0,5] \).

  14. Domanda 14
    Se le dimensioni \( L \),\( W \) e \( H\) di una scatola rettangolare cambiano alla velocità \( \dfrac{dL}{dt} = 0.1 \; cm/sec \), \ ( \dfrac{dW}{dt} = - 0.2 \; cm/sec\) e \( \dfrac{dH}{dt} = 0.3 \; cm/sec\), a quale velocità è il volume della scatola cambiando quando \( L = 20 \; cm \), \( W = 8 \; cm \) e \( H = 5 \; cm \)?

  15. Domanda 15
    Quali sono le dimensioni del rettangolo di area maggiore inscrivibile in un semicerchio di raggio 3?

    diagram of circle and rectangle for question 15

Soluzioni dettagliate alle domande di cui sopra


  1. Soluzione alla domanda 1
    Affinché la funzione \( f \) assuma valori reali, l'espressione sotto il radicale al numeratore deve essere non negativa e l'espressione sotto il radicale al denominatore deve essere positiva; da qui le disuguaglianze da risolvere
    \( x - 1 \ge 0 \) e \( 4 - x^2 \gt 0 \)
    Gli insiemi di soluzioni per la prima e la seconda disuguaglianza sono rispettivamente
    \( x \ge 1 \) e \( -2 \lt x \lt 2 \)
    Entrambe le disuguaglianze devono essere soddisfatte simultaneamente, quindi il dominio della funzione data è l'intersezione degli insiemi \( x \ ge 1 \) e \( -2 \lt x \lt 2 \) che è dato da
    \( 1 \le x \lt 2 \)


  2. Soluzione alla domanda 2
    Secondo le proprietà delle funzioni inverse, un modo per trovare le intervallo della funzione data è trovare il dominio della sua inversa.
    Proviamo prima che \( f \) è una funzione uno a uno e poi troviamo la sua inverso. Per dimostrare che \( f \) è una funzione uno a uno e quindi invertibile, usiamo il contropositivo della funzione uno a uno e iniziamo con \( f(a) = f(b) \) e prova che \( a = b \). Quindi
    \( \dfrac{a - 1}{2-3 a} = \dfrac{b - 1}{2-3 b} \)
    Moltiplicazione incrociata
    \( (a - 1)(2-3 b) = (2 - 3 a)(b - 1) \)
    Espandere
    \( 2a - 2 - 3 a b + 3 b = 2 b - 2 - 3 ab + 3 a \)
    Raggruppa come termini
    \( 2a = 2b\)
    Risolvi per a
    \(a = b\)
    il che dimostra che la funzione \( f \) è una funzione uno a uno e quindi ha un'inversa
    L'inverso di \( f \) può essere calcolato partendo dall'equazione
    \( y = \dfrac{x - 1}{2-3x} \)
    Moltiplicazione incrociata dell'equazione precedente
    \( 2 y - 3 x y = x - 1 \)
    e risolvi per \( x \)
    \( x = \dfrac{2 y + 1}{3y + 1} \)
    Scambia \( x \) e \( y \) nell'equazione precedente per ottenere l'inverso \( f^{-1} \)
    \( f^{-1}(x) = y = \dfrac{2 x + 1 }{3x + 1} \).
    La domiana di \( f^{-1} \) è l'insieme di tutti i numeri reali tranne \( -\dfrac{1}{3} \). Quindi l'intervallo di \( f \) è l'insieme di tutti i numeri reali tranne \( -\dfrac{1}{3} \) che può essere scritto in forma di intervallo come
    \( (-\infty , - \dfrac{1}{3}) \cup (- \dfrac{1}{3} , +\infty) \)


  3. Soluzione alla domanda 3
    Scrivi la funzione come equazione come segue
    \( y = ln (2x - 3) + 2 \)
    Risolvi quanto sopra per x
    \(y - 2 = ln (2x - 3) \)
    \( 2x - 3 = e^{y - 2} \)
    \( 2 x = e^{y - 2} + 3 \)
    \( x = \dfrac{1}{2} (e^{y - 2} + 3) \)
    Interscambio \( x \) e \( y \)
    \( y = \dfrac{1}{2} (e^{x - 2} + 3) \)
    L'inverso di \( f \) è dato da
    \( f^{-1}(x) = \dfrac{1}{2} (e^{x - 2} + 3) \)


  4. Soluzione alla domanda 4
    a)
    Il limite è nella forma indeterminata \( \dfrac{0}{0} \).
    Moltiplica numeratore e denominatore per il coniugato del numeratore \( -\dfrac{1}{\sqrt x} - \dfrac{1}{4} \)

    \( \lim_{x\to 16} \dfrac{-\dfrac{1}{\sqrt x} + \dfrac{1}{4}}{x - 16 } \) = \( \lim_{x\to 16} \dfrac{ (-\dfrac{1}{\sqrt x} + \dfrac{1}{4})(-\dfrac{1}{\sqrt x} - \dfrac{1}{4}) } { (x - 16)(-\dfrac{1}{\sqrt x} - \dfrac{1}{4}) } \)
    Semplificare
    \( = \lim_{x\to 16} \dfrac{ \dfrac{1}{x} - \dfrac{1}{16} }{ (x - 16)(-\dfrac{1}{\sqrt x} - \dfrac{1}{4}) } = \lim_{x\to 16} \dfrac{ \dfrac{16-x}{16x} }{ (x - 16)(-\dfrac{1}{\sqrt x} - \dfrac{1}{4}) } \)

    \( = \lim_{x\to 16} \dfrac{-1}{16x(-\dfrac{1}{\sqrt x} - \dfrac{1}{4})} = \dfrac{-1}{ 16 \times 16(-\dfrac{1}{\sqrt 16} - \dfrac{1}{4})} = \dfrac{1}{128}\)

    b)
    Il limite è della forma indeterminata \( \dfrac{\infty}{\infty} \).
    Dividi tutti i termini al numeratore e tutti i termini al denominatore per con il termine con la massima potenza che in \( x^4 \)
    \( \lim_{x\to +\infty} \dfrac{-x^3+2x-1}{x^4 - 3 x^3 + 9 } = \lim_{x\to +\infty} \dfrac{ \dfrac{-x^3}{x^4}+\dfrac{2x}{x^4}-\dfrac{1}{x^4}}{\dfrac{x^4}{x^4} - \dfrac{3 x^3}{x^4} + \dfrac{9}{x^4} }\).
    Semplifica i termini razionali
    \( = \lim_{x\to +\infty} \dfrac{\dfrac{-1}{x}+\dfrac{2}{x^3}-\dfrac{1}{x^4}}{1 - \dfrac{3}{x} + \dfrac{9}{x^4} } = \dfrac{0+0-0}{1 - 0 + 0 } = \dfrac{0}{1} = 0\)

    c)
    Il limite è della forma indeterminata \( \infty \cdot 0 \).
    Sia \( t = \dfrac{3}{x} \) e riscriviamo il limite in termini di t.
    \( \lim_{x\to +\infty} x \sin(\dfrac{3}{x}) = \lim_{t\to 0} 3 \dfrac {\sin(t)}{t} \).
    Usando il noto risultato \( \lim_{t\to 0} \dfrac {\sin(t)}{t} = 1 \), il limite vale
    \( = 3 × 1 = 3\)

    d)
    \( \lim_{x\to 0} \dfrac{\sin(x)+x}{2x^2+x} = \dfrac{0}{0}\), forma indeterminata
    Usa la Regola di L'Hospital nella forma indeterminata, possiamo scrivere
    \( \lim_{x\to 0} \dfrac{\sin(x)+x}{2x^2+x} = \lim_{x\to 0} \dfrac{ d (\sin(x)+x) / dx }{ d(2x^2+x)/dx} = \lim_{x\to 0} \dfrac{ cos(x)+ 1 }{ 4x+1} = \dfrac{ cos(0)+ 1 } { 4× 0+1} = 2\)

    e)
    \( \lim_{x\to +\infty} \dfrac{\sin(x)+1}{x} \).
    È ben noto che l'intervallo di \( \sin(x) \) è dato da
    \( -1 \le \sin(x) \le 1 \)
    Aggiungi 1 a tutti i termini della disuguaglianza per ottenere la seguente disuguaglianza
    \( -1 + 1 \le \sin(x) + 1 \le 1 + 1 \)
    \( 0 \le \sin(x) + 1 \le 2 \)
    Dividi tutti i termini della disuguaglianza di cui sopra per \( x \) positivo
    \( \dfrac{0}{x} \le \dfrac{\sin(x) + 1}{x} \le \dfrac{2}{x} \)
    Abbiamo \( \lim_{x\to +\infty} \dfrac{0}{x} = 0 \) e \( \lim_{x\to +\infty} \dfrac{2}{x} = 0 \ )
    Utilizzando il teorema della squeezing (o Sandwich) , possiamo valutare il limite dato come segue
    \( \lim_{x\to +\infty} \dfrac{\sin(x)+1}{x} = 0 \)


  5. Soluzione alla domanda 5
    a)
    Il grafico di \( y = e^{x-1} \) e \( y = x \) è mostrato sotto. La derivata di \( y = e^{x-1} \) è uguale a \( y ' = e^{x-1} \) e la pendenza \( m \) della tangente in \( x = 1 \) è il valore della derivata in \( x = 1\). Quindi

    grafici di y = e^(x-1) e y = x
    \( m = e^{1-1}= 1 \)
    L'equazione della retta tangente nel punto \( (1,1) \) è data da
    \( y - 1 = 1 \times (x - 1) \)
    Il che semplifica a
    \(y = x\)
    Quindi i grafici di \( y = e^{x-1} \) e \( y = x \) sono tangenti nel punto \( (1,1) \) e possiamo quindi affermare graficamente che \( e^ {x-1} \ge x \).
    b)
    Le derivate prima e seconda di \( f \) sono date da
    \( f'(x) = \dfrac{x^2}{2}-e^{x-1} \)
    \( f''(x) = x-e^{x-1} \)
    Un punto di flesso si verifica a un valore di \( x \) dove \( f''(x ) \) cambia segno. Abbiamo visto nella parte a) che \( e^{x-1} \ge x \) che può essere scritto come
    \( x-e^{x-1} \le 0 \)
    e quindi \( f''(x) \) è negativo e ha uno zero in \( x = 1 \). Quindi il grafico di \( f(x) \) è concavo verso il basso e non ha un punto di flesso perché \( f''(x) \) non cambia segno.


  6. Soluzione alla domanda 6
    a) La regola della somma delle derivate dà: \( f'(x) = e^{x-1 } + \dfrac{3}{3x-1} + 2 \cos(2x+1) \)
    b) Regola del prodotto delle derivate: \( g'(x) = 4(2x-1)(\tan(x)-1) + (2x-1)^2(\sec^2(x)) \)
    c) regola del quoziente delle derivate: \( h'(x) = \dfrac{ (1 + \sin(x))(x^2-2x+1) - (x - \cos(x))(2x-2 ) }{(x^2-2x+1)^2} \)

    d) Sia \( u = \sqrt{x^3 - \dfrac{1}{x} + 2} \) , scrivi la funzione \(m \) come \(m = \sin u \) quindi usa regola della catena delle derivate

    \( m'(x) = \dfrac{d m}{d u} \dfrac{d u}{d x} = \cos(u) \dfrac{1}{2}(3x^2+\dfrac{1}{x ^2})(x^3 - \dfrac{1}{x} + 2)^{-1/2} \)

    \( = \dfrac{1}{2} \cos \left(\sqrt{x^3 - \dfrac{1}{x} + 2} \right) (3x^2+\dfrac{1}{x^ 2})(x^3 - \dfrac{1}{x} + 2)^{-1/2} \)

    e)
    Riscrivi \( 3^{ 2x+3} \) e \( \log_3(2x-1) \) come
    \( 3^{2x+3} = e^{(2x+3) \ln 3}\) , cambio di base degli esponenziali
    \( \log_3(2x-1) = \dfrac{ \ln(2x-1)}{ \ln 3}\) , cambio di base dei logaritmi
    Sostituisci e riscrivi \( n(x) \) come
    \( n(x) = 3^{ 2x+3} + \log_3(2x-1) = e^{(2x+3) \ln 3} + \dfrac{ \ln(2x-1)}{ \ln 3} \)
    Calcoliamo ora la derivata
    \( n'(x) = ( 2 \ln 3 ) e^{(2x+3) \ln 3} + \dfrac{1}{ \ln 3} \dfrac{2}{2x-1} = ( 2 \ln 3 ) 3^{2x+3} + \dfrac{2}{\ln 3(2x-1)} \)


  7. Soluzione alla domanda 7
    Prima distinguiamo l'equazione data implicitamente
    \( 2 y \dfrac{d y}{d x} cos(y^2) = 2 x \)
    \( \dfrac{d y}{d x} = \dfrac{ x}{ y cos(y^2)} \)
    La pendenza \( m \) della retta tangente è data dal valore di \( \dfrac{d y}{d x} \) nel punto \( ( 0,\sqrt{\pi}) \).
    \( m = \dfrac{(0)}{\sqrt{\pi} cos((\sqrt{{\pi}})^2)} = 0 \)
    L'equazione, in forma di pendenza puntuale, della retta tangente alla curva nel punto \( ( 0,\sqrt{\pi}) \) è data da
    \( y - \sqrt{\pi} = 0(x - 0) \)
    È una linea orizzontale data da
    \(y = \sqrt{\pi} \)


  8. Soluzione alla domanda 8
    \( f(x) \) è continuo sugli intervalli \( (-\infty , 1) \) , \( (1,2) \) e \( (2 , +\infty) \). Dobbiamo trovare \( a \) e \( b \) in modo che sia continua anche in \( x = 1 \) e \( x = 2 \) e quindi continua in \( (-\infty , +\ infty ) \).
    \( f(1) = 1 \)
    \( \lim_{x\to 1^-} f(x) = 1 \)
    \( \lim_{x\to 1^+} f(x) = a(1)^3+b = a + b \)
    I limiti da sinistra e da destra di \( 1 \) devono essere uguali
    \( a + b = 1 \) (equazione 1)
    \( f(2) = 2 + 2b\)
    \( \lim_{x\to 2^-} f(x) = a(2)^3 + b = 8 a + b \)
    \( \lim_{x\to 2^+} f(x) = 2 + 2 b \)
    I limiti da sinistra e da destra di \( 2 \) devono essere uguali
    \( 8 a + b = 2 + 2 b \) (equazione 2)
    Risolvi le equazioni (1) e (2) simultaneamente per trovare
    \( a = \dfrac{1}{3} \) e \( b = \dfrac{2}{3} \)


  9. Soluzione alla domanda 9
    Trova la derivata di \( y \).
    \(y' = 1 + \cos(x) \)
    La pendenza \( m \) della tangente in \( x = 0 \) è uguale al valore di \( y' \) in \( x = 0 \). Quindi
    \( m = 1 + \cos(0) = 2 \)
    La coordinata y del punto di tangenza \( P \) è data dal valore di \( y \) in \( x = 0 \). Quindi
    \( P(0 , 0 + \sin(0)) = P(0,0) \)
    L'equazione della retta tangente in forma di pendenza puntuale è data da
    \(y - 0 = 2(x - 0) \)
    e in forma di intercettazione del pendio
    \(y = 2 x \)


  10. Soluzione alla domanda 10
    La definizione della derivata \( f' \) della funzione \( f \) è data dal limite
    \( f'(x) = \lim_{h\to 0} \dfrac{f(x+h)-f(x)}{h} \)
    Sostituisci \( f(x) \) con \( \sqrt{x+2} \) nella definizione precedente per ottenere
    \( f'(x) = \lim_{h\to 0} \dfrac{\sqrt{x + h +2}- \sqrt{x+2} }{h} \)
    Il limite di cui sopra è della forma indeterminata \( \dfrac{0}{0} \). Moltiplica il numeratore e il denominatore per il coniugato del numeratore
    \( f'(x) = \lim_{h\to 0} \dfrac{ (\sqrt{x + h +2}- \sqrt{x+2} ) (\sqrt{x + h +2} + \ sqrt{x+2} ) }{h(\sqrt{x + h +2} + \sqrt{x+2} )} \)
    Espandi il numeratore e semplifica
    \( f'(x) = \lim_{h\to 0} \dfrac{ (x + h + 2)- (x + 2) ) }{h(\sqrt{x + h +2} + \sqrt{ x+2} )} = \lim_{h\to 0} \dfrac{ h }{h(\sqrt{x + h +2} + \sqrt{x+2} )}\)
    Dividi numeratore e denominatore per \( h \) (o cancella \( h \))
    \( f'(x) = \lim_{h\to 0} \dfrac{ 1 }{\sqrt{x + h +2} + \sqrt{x+2} }\)
    Valutare il limite e quindi la derivata
    \( f'(x) = \dfrac{ 1 }{ \sqrt{x + 0 + 2} + \sqrt{x + 2} } = \dfrac{ 1 }{2 \sqrt{x + 2} }\)



  11. Soluzione alla domanda 11
    Trova le derivate prima e seconda
    \( f'(x) = e^x (x^2-5x+8) + e^x (2 x -5) +\dfrac{x^3}{3}-\dfrac{x^2}{ 2} = e^x (x^2-3x+3)+\dfrac{x^3}{3}-\dfrac{x^2}{2} \)
    \( f''(x) = e^x(x^2-3x+3)+e^x(2x-3) + x^2-x = e^x(x^2-x)+x^ 2-x = x (x - 1) e^x \)
    \( f'' \) ha due zeri: \( x = 0 \) e \( x = 1\) e \( e^x \) è sempre positivo. Quindi la tavola dei segni di \( f'' \) ha tre intervalli
    1) \( (-\infty , 0 ) \) , valore di prova \( x = -1 \) , \( f''(-1) = 2/e \) , quindi \( f''(x) \) è positivo sull'intervallo \( (-\infty , 0 ) \).
    2) \( (0 , 1 ) \) , valore test \( x = 1/2 \) , \( f''(1/2) = -\dfrac{\sqrt 2}{4} \) , quindi \( f''(x) \) è negativo sull'intervallo \( (0 , 1 ) \).
    3) \( (1 , +\infty ) \) , valore test \( x = 2 \) , \( f''(2) = 2e^2 \) , \( f''(x) \) è positivo sull'intervallo \( (1 , +\infty ) \).
    \( f'' \) è concava in alto sugli intervalli \( (-\infty , 0 ) \) e \( (1 , +\infty ) \) , e concava verso il basso sull'intervallo \( (0 , 1 ) \).
    \( f'' \) cambia segno in \( x = 0 \) e \( x = 1 \) e quindi ha punti di flesso in \( x = 0 \) e \( x = 1 \).



  12. Soluzione alla domanda 12
    Il metodo di Newton si basa sul seguente algoritmo: conoscere un'approssimazione \( x_n \) alla soluzione di un equazione \( f(x) = 0 \), l'approssimazione successiva \( x_{n+1}\) dell'equazione è data da
    \( x_{n+1} = x_n - \dfrac{f(x_n)}{f'(x_n)} \).
    La soluzione dell'equazione data \( e^x = x^3 \) è uguale alla soluzione dell'equazione \( f(x) = e^x - x^3 = 0 \)
    \( f'(x) = e^x - 3x^2 \)
    Conosciamo una prima approssimazione \( x_1 = 2 \); usando l'algoritmo di Newton approssimiamo \( x_2\) per
    \( x_{2} = x_1 - \dfrac{f(x_1)}{f'(x_1)} = 2 - \dfrac{e^2 - 2^3}{e^2 - 3 \times 2^2} \approx 1,87 \) (arrotondato alla seconda cifra decimale)
    Un'approssimazione \(x_3\) può essere ottenuta usando \( x_2 \) trovata sopra e così via.



  13. Soluzione alla domanda 13
    Trova la derivata prima e fattorizzala.
    \( f'(x) = 4 x^3 - 3 x^2 = x^2(4x - 3)\)
    \( f'(x) \) ha due zeri in \( x = 0 \) e \( x = 3/4 \) ed entrambi sono all'interno dell'intervallo \( [0,5] \). Gli zeri di \( f'(x) \) sono chiamati punti critici.
    Valutiamo ora la funzione agli estremi dell'intervallo dato e agli zeri di \( f'(x) \).
    \( f(0) = 0 \)
    \( f(5) = 5^4 - 5^3 = 500 \)
    \( f(3/4) = (3/4)^4 - (3/4)^3 = -\dfrac{27}{256} \)
    Confrontando questi valori, \( f(x) \) ha un massimo assoluto pari a \(500 \) all'estremo \( x = 5 \) e un minimo assoluto pari a \( -\dfrac{27}{256} \) nel punto critico \( x = 3/4 \)
    Sono incluse altre domande sul minimo e massimo assoluto di una funzione.


  14. Soluzione alla domanda 14
    Il volume \( V \) della scatola rettangolare di dimensioni \( L \),\( W \) e \( H\) è dato da
    \( V = L(t) W(t) H(t) \)
    dove \( L(t)\) , \( W(t) \) e \( H(t) \) sono funzioni del tempo \( t \).
    Poiché le dimensioni della scatola cambiano nel tempo, anche il volume della scatola cambia nel tempo al tasso di cambiamento di \( V\) è dato dalla derivata prima \( \dfrac{dV}{dt} \).
    \( \dfrac{dV}{dt} = W(t) H(t) \dfrac{d L}{dt } + L(t) H(t) \dfrac{d W}{dt } + L(t ) W(t) \dfrac{d H}{dt } \)
    Sostituisci le quantità note con i loro valori numerici
    \( \dfrac{dV}{dt} = 8 \times 5 \times 0.1 + 20 \times 5 \times (-0.2) + 20 \times 8 \times 0.3 = 32 \; cm^3 / sec \)



  15. Soluzione alla domanda 15
    Equazione di un cerchio con raggio \( 3 \) e centro in \( (0 , 0) \)dato di
    \( x^2 + y^2 = 3^2 \)
    Risolvi l'equazione precedente fo y
    \(y^2 = 9 - x^2\)
    \(y = \pm \sqrt{9 - x^2} \)
    L'equazione per il semicricolo superiore è \(y = \sqrt(9-x^2) \)
    Un punto sul semicerchio con coordinata x \( x \) ha una coordinata y uguale a \(\sqrt{9 - x^2} \) (vedi grafico sotto)
    Il rettangolo ha una lunghezza \( L = 2x \) e una larghezza (o altezza) \( W = \sqrt(9-x^2) \). L'area \( A \) del rettangolo è data da
    \( A(x) = L \times W = 2 x \sqrt(9-x^2) \) , \( 0 \le x \le 3 \)
    diagramma di cerchio e rettangolo per la soluzione della domanda 15
    Trova la derivata prima di \( A \)
    \( \dfrac{d A}{dx} = 2\sqrt{9-x^2} + (2x) (\dfrac{1}{2}) (-2x) (9-x^2)^{- 1/2} =\dfrac{2\left(-2x^2+9\right)}{\sqrt{9-x^2}} \)
    Gli zeri di \( \dfrac{d A}{dx} \) sono dati dagli zeri del numeratore di \( \dfrac{d A}{dx} \)
    \(-2x^2+9 = 0 \)
    Restituisce due zeri: \( x = \sqrt{4.5} \) e \( x = - \sqrt{4.5} \)
    Consideriamo lo zero in \( x = \sqrt{4.5} \) poiché è all'interno dell'intervallo \( [0 , 3 ] \)
    Valutiamo ora l'area \( A(x) \) agli estremi \( x = 0 \) e \( x = 3 \) e nel punto critico \( x = \sqrt{4.5} \).
    \( A(0) = 0 \)
    \( A(3) = 0 \)
    \( A(\sqrt{4.5}) = 9 \)
    L'area A è massima è per \( x = \sqrt{4.5} \)
    Le dimensioni sono:
    \( L = 2 x = 2 \sqrt{4.5} \approx 4.24 \)
    \( W = \sqrt(9-x^2) = \sqrt(9-\sqrt{4.5}^2) \approx 2.12 \)
    Altri problemi di ottimizzazione inclusi.

Altri riferimenti e collegamenti

Problemi di calcolo